Gönderen Konu: Diyafont Denklemler Çalışma Soruları ($138$ Tane)  (Okunma sayısı 18151 defa)

Çevrimdışı AtakanCİCEK

  • G.O Demirbaş Üye
  • ******
  • İleti: 264
  • Karma: +4/-0
  • Manisa
Diyafont Denklemler Çalışma Soruları ($138$ Tane)
« : Temmuz 02, 2019, 03:15:29 ös »
$1.$ aşama $2.$ aşama ve uluslararası matematik olimpiyatları düzeyinde Diyafont Denklem soruları seçtim. Çözümleri de isimlerinizle beraber paylaşırsak sevinirim. Pdf haline getiriyorum.


NOT:Siteye PDF eklemeyi bilmediğim için bu şekilde gönderiyorum.


NOT 2: $78.$ soru ve $99$'den sonraki sorular daha çözülmemiştir ($99$ dahil, $102$, $104$, $105$ $109$, $110$, $113$, $118$, $134$, $136$ hariç).


$1)$ $\frac{x^3+y^3+z^3}{3}=xyz+27$ denklemini $x<y<z$ için pozitif tam sayılarda çözünüz.

$2)$ $n^4+3n^2+1$ sayısının hiçbir $n$ pozitif tam sayısı için tamkare olmadığını gösteriniz.(UMO $2.$ Aşama $1992$)

$3)$ $y^2=x^3+7$ denklemini tam sayılarda çözünüz.

$4)$ $k!+48=48.(k+1)^m$ olacak şekilde $k$ ve $m$ negatif olmayan tam sayılarının bulunmadığını gösteriniz.(Kanada Matematik Olimpiyatı)

$5)$ $2^m-3^n=7$ olacak şekilde tüm $(m,n)$ ikililerini bulunuz.(Avusturya-Polonya Matematik Olimpiyatı $1993$)

$6)$ $a^2+b=b^{1999}$ denklemini tam sayılarda çözünüz.(Estonya Matematik Olimpiyatı $1999$)

$7)$ $a^3+b^3+c^3=2001$ denklemini sağlayan kaç tane $(a,b,c)$ pozitif tam sayı üçlüsü vardır?($Balkan Junior 2001$)

$8)$ $5.(xy+xz+yz)=3xyz$ denkleminin pozitif tam sayılarda çözümlerini bulunuz.

$9)$ $x^3-y^3=xy+61$ denklemini pozitif tam sayılarda çözünüz.(Sovyet Matematik Olimpiyatı $1981$)

$10)$ $x^3+9xy+127=y^3$ denklemini sağlayan kaç tane $(x,y)$ tam sayı çifti vardır?

$11)$ $n^3+7n-133$ ifadesi pozitif bir tam sayının kübü ise $n$ sayısına ''iyi sayı'' diyelim. Tüm iyi sayıları bulunuz.(USC Math Contest)

$12)$ $n^2-19n+99$ sayısı tamkare olacak şekildeki tüm $n$ tam sayılarının toplamı kaçtır?(AIME $1999$)

$13)$ $n^4+n^3+1$ tamkare olacak şekilde tüm $n$ tam sayılarını bulunuz.

$14)$ $5n^2=36a^2+18b^2+6c^2$ denklemini tam sayılarda çözünüz.(Asya-Pasifik Matematik Olimpiyatı$1989$)

$15)$ $10x^3+20y^3+8xyz=1999z^3$ denkleminin tam sayılarda kaç çözümü vardır?(Municipial $1999$)

$16)$ $(m-n)^2=\frac{4mn}{m+n-1}$ denklemini sağlayan $0<m+n<100$ olacak şekilde kaç $(m,n)$ tam sayı çifti vardır?(Estonya Matematik Olimpiyatı $1999$)

17)Aşağıdaki denklem sistemini pozitif tam sayılar kümesinde çözünüz.(İsveç Matematik Olimpiyatı $1984$)\begin{equation*}
\begin{cases}
a^3-b^3-c^3=3abc,
\\
a^2=2.(a+b+c)
\end{cases}
\end{equation*}
 
$18)$ $2n^3-m^3=mn^2+11$ denklemini tam sayılarda çözünüz.(İsveç Matematik Olimpiyatı $1994$)

$19)$ $x,y,\frac{x^2+y^2+6}{xy}$ sayıları tam sayı ise $\frac{x^2+y^2+6}{xy}$ sayısının tamküp olduğunu gösteriniz.(Estonya Matematik Olimpiyatı $1995$)

$20)$ $m,n$ tam sayılar, $p$ bir asal sayı olmak üzere $\frac{13^m+p.2^n}{13^m-p.2^n}$ bir pozitif tam sayı olacak şekilde kaç $(m,n,p)$ üçlüsü vardır?(UMO $1.$ Aşama $2018$)

$21)$ $x$ bir asal sayı olmak üzere $x-y^4=4$ eşitliğini tam sayılarda çözünüz.

$22)$ $x^6+3x^3+1=y^4$ olacak şekilde $(x,y)$ ikililerini bulunuz.(Romanya Matematik Olimpiyatı)

$23)$ $a,b,c$ tam sayı ve $1<a<b<c$ olmak üzere, (a-1).(b-1).(c-1) in abc-1 in bölenlerinden biri olmasını sağlayan $(a,b,c)$ üçlülerini bulunuz.(IMO $1992$)

$24)$ $(x+1)^4-(x-1)^4=y^3$ eşitliğini sağlayan tüm $(x,y)$ tam sayı ikililerini bulunuz.(Avusturya Matematik Olimpiyatı)

$25)$ $p^3-q^5=(p+q)^2$ eşitliğini asal sayılar için çözünüz.(Rusya Matematik Olimpiyatı)

$26)$ $x^2-y!=2001$ denklemini pozitif tam sayılarda çözünüz.

$27)$ $5^x.7^y+4=3^z$ denklemini negatif olmayan tam sayılarda çözünüz.(Bulgaristan Matematik Olimpiyatı)

$28)$ $x^3-4xy+y^3=-1$ eşitliğini sağlayan tüm tam sayı değerlerini bulunuz.

$29)$ $x^2-y^2=2xyz$ denklemini pozitif tam sayılarda çözünüz.

$30)$ $x^2+axy+y^2=z^2$ denkleminin $(x,y,z)$ için genel çözümlerini bulunuz .

$31)$ $xy.(x^2+y^2)=2z^2$ denklemini tam sayılarda çözünüz.

$32)$ $ p^3+p^2+11p+2=q$ denklemini asal sayılarda çözünüz.

$33)$ $x^3-y^3=2y^2+1$ denklemini tam sayılarda çözünüz.

$34)$ $x^4+4^x=p$ denklemini $p$ asal sayısı ve $x$ tam sayısı için çözünüz.

$35)$ $c^2+1=(a^2-1).(b^2-1)$ denkleminin bütün çözümlerini bulunuz.

$36)$ $9^x-3^x=y^4+2y^3+y^2+2y$ denkleminin tam sayı çözümlerini bulunuz.

$37)$ $x$ ve $y$ $5$' ten büyük asal çarpanı olmayan sayılar olmayan pozitif sayılar olmak üzere $k\ge0$ olan bir $k$ tam sayısı için $x^2-y^2=2^k$ denklemini çözünüz.

$38)$ $n^2+3^n$ sayısını tamkare yapan bütün pozitif tam sayıları bulunuz.

$39)$ $a,b,m,n$ pozitif tam sayılardır. $n>1$ için  $a^n+b^n=2^m$ ise $a=b$ olduğunu kanıtlayınız.

$40)$ $2^m+3^n=k^2$ denklemini sağlayan pozitif tam sayıları bulunuz.

$41)$ $p^m.q^n=(p+q)^2+1$ olacak şekildeki tüm $(m,n,p,q)$ pozitif tam sayılarını bulunuz.

$42)$ $ab\neq1$ $a,b\ge0$ olmak üzere  $k=\dfrac{a^2+ab+b^2}{ab-1}$ ifadesi ile elde edilebilecek tüm $k\ge0$ değerlerini bulunuz.

$43)$ $x^{2006}-4y^{2006}-2006=4y^{2007}+2007y$ denkleminin pozitif tam sayılarda kaç çözümü vardır?

$44)$ $n\ge3$ koşulunu sağlayan her doğal sayı için $7x_n^2+y_n^2=2^n$ denklemini sağlayacak $x_n,y_n$  tek  bir doğal sayı ikilisinin bulunduğunu gösteriniz.

$45)$ $\varphi(n)=\varphi(2n)$ denklemini pozitif tam sayılarda çözünüz.($\varphi(n)$, $1$ ile $n$ arasında ($n$ dahil), $n$'den küçük $n$ ile aralarında asal olan pozitif tam sayıların sayısını belirtmektedir.)

$46)$ $x^4+y^4=z^2$ denkleminin pozitif tam sayılar kümesinde  çözümünün olmadığını gösteriniz.

$47)$ $a!+b^3=18+c^3$ denklemini pozitif tam sayılarda çözünüz.(UMO $1.$ Aşama $2013$)

$48)$ $m^3-n^3=9^k+123$ eşitliğini sağlayan kaç $(m,n,k)$ negatif olmayan tam sayı üçlüsü vardır?(UMO $1.$ Aşama $2014$)

$49)$ $p$ , $4p^2+1$ $6p^2+1$  birer asal sayı olacak şekildeki $p$ sayıları için $2x^3-y^3=p$ denklemini pozitif tam sayılarda çözünüz.

$50)$ $t^2+1=s.(s+1)$ denklemini tam sayılarda çözünüz.(UMO $2.$ Aşama $1994$)

$51)$ $x^3+3367=2^n$ denklemini pozitif tam sayılarda çözünüz.(UMO $2.$ Aşama $1998$)

$52)$ $3^x+11^y=z^2$ denklemini pozitif tam sayılarda çözünüz.(UMO $2.$ Aşama $2001$)

$53)$ $5^m+7^n=k^3$ eşitliğini sağlayan tüm $(m,n,k)$ negatif olmayan tam sayı üçlülerini bulunuz.(UMO $2.$ Aşama $2005$)

$54)$ $k>1$ bir sayı ve $p=6k+1$ bir sayı ve $m=2^p-1$ ise $\frac{2^{m-1}-1}{127m}$ sayısının bir tam sayı olduğunu gösteriniz.(UMO $2.$ Aşama $2007$)

$55)$ $2^n+n=m!$ denklemini $(m,n)$ pozitif tam sayıları için çözünüz.(UMO $2.$ Aşama $2013$)

$56)$ $x^3=3^y.7^z+8$ eşitliğini sağlayan $(x,y,z)$ pozitif tam sayı üçlülerini bulunuz.(UMO $2.$ Aşama $2014$)

$57)$ $m,n$ pozitif tam sayılar olmak üzere, $k=\dfrac{(m+n)^2}{4m.(m-n)^2+4}$ ifadesi tam sayı ise $k$ nın tamkare olduğunu gösteriniz.(UMO $2.$ Aşama $2015$)

$58)$ $a^2+b^2+c^2+d^2=a^2b^2c^2d^2$ denkleminin pozitif tam sayılarda kaç çözümü vardır?(UMO TSÇ $1991$)

$59)$ $a^2+b^2+3=abc$ olacak şekilde $(a,b,c)$ çözümlerini bulunuz.(UMO TSÇ $1994$)

$60)$ $n$ pozitif bir tam sayı olduğuna göre $x^2-xy+y^2=n$ denklemini sağlayan $(x,y)$ pozitif tam sayı sıralı ikililerinin sayısının $3$ ile bölündüğünü ispatlayınız.(UMO TSÇ $2000$)

$61)$ $5^x=1+4y+y^4$ eşitliğini sağlayan $(x,y)$ tam sayı ikililerini bulunuz.(UMO TSÇ $2001$)

$62)$ $m^6=n^{n+1}+n-1$ eşitliğini sağlayan $(m,n)$ pozitif tam sayı ikililerini bulunuz.(UMO TSÇ $2013$)

$63)$ $l,m,n$ pozitif tam sayılar ve $p$ bir asal sayı olmak üzere, $p^{2l-1}.m.(mn+1)^2+m^2$ bir tamkare ise $m$ nin de bir tamkare olduğunu ispatlayınız.(UMO TSÇ $2015$)

$64)$ Ondalık yazılımındaki rakamların çarpımı $x^2-10x-22$ ye eşit olan tüm $x$ doğal sayılarını bulunuz.(IMO $1968$)

$65)$ $2^n+1=n^2m$ denklemini $m$ tam sayıları ve $n>1$ tam sayıları için çözünüz.(IMO $1990$)

$66)$ $\dfrac{n^3+1}{mn-1}$ sayısının bir tam sayı olmasını sağlayan tüm $(m,n)$ sıralı pozitif tam sayı ikililerinin sayısını bulunuz.(IMO $1994$)

$67)$ $\dfrac{a^2}{2ab^2-b^3+1}$ bir pozitif tam sayı olacak şekilde kaç $(a,b)$ pozitif tamsayısı vardır?(IMO $2003$)

$68)$ $1+2^x+2^{2x+1}=y^2$ denklemini tam sayılarda çözünüz.(IMO $2006$)

$69)$ $k,n$ negatif olmayan tam sayılar ve $p$ bir asal sayı ise $5^k-3^n=p^2$ denklemini çözünüz.

$70)$ $m^4+2n^3+1=mn^3+n$ eşitliğini sağlayan tüm $(m,n)$ tam sayı ikililerini bulunuz.(Avr.Kızlar.mat.olim.TSÇ $2015$)

$71)$ $k!=(2^n-1).(2^n-2).(2^n-4)...(2^n-2^{n-1})$ denklemini sağlayan $(k,n)$ pozitif tam sayı ikililerini bulunuz.

$72)$ $7n^2=m^3+15m$ denkleminin tam sayılarda kaç çözümü vardır ? (İSBO $2019$ Ortaokul)

$73)$ $a>1$ ve $b>1$ tam sayıları için $a^{b^2}=b^a$ denklemini çözünüz.

$74)$  $2^x=3^y+5$ denklemini tam sayılarda çözünüz.  (IMO $1959-1966$ LongList)

$75)$ $x^2+y^2=(x-y)^3$  diyafont denkleminin tüm tam sayı çözümlerini bulunuz. (IMO $1971$ LongList)

$76)$ $1+x+x^2+x^3+x^4=y^4$ denklemini tamsayılarda çözünüz. (IMO $1972$  LongList)

$77)$ $p^3+m.(p+2)=m^2+p+1$ denklemini $p$ asal sayıları ve $m$  pozitif tam sayıları için çözünüz. ($AOPS$)

$78)$ $2a^4-2a^2=b^2-1$  denklemini pozitif tam sayılarda çözünüz. ($AOPS$)

$79)$ $ab+ac+bc=1$  eşitliğini sağlayan $a,b,c$ tam sayıları için $(a^2+1).(b^2+1).(c^2+1)=3^x-5^y$ eşitliğini sağlayan $x,y$ tam sayılarının bulunmasını sağlayan kaç $(a,b,c)$ üçlüsü vardır?

$80)$ $(x^2+1).(y^2+1)+2.(x-y).(1-xy)=n^2+4xy$ eşitliğini tam sayılarda çözüm sayısı kaçtır?

$81)$ $\dfrac{1}{x}+\dfrac{1}{y}=\dfrac{1}{n}$ eşitliğinin pozitif tam sayılarda çözüm sayısı kaçtır?

$82)$ $(xy-9)^2=x^2+y^2$ denklemini negatif olmayan tam sayılarda çözünüz (Hint Olimpiyatından değiştirilmiştir.)

$83)$ $x^2.(y-1)+y^2(x-1)=1$ denklemini tam sayılarda çözünüz. (Polonya Matematik Olimpiyatı)

$84)$ $x^4+4=py^4$ eşitliğini tam sayılarda çözümlü yapan tüm $p$ asallarını bulunuz. (Ion Cucurezeanu)

$85)$ $k_1+k_2+k_3+...+k_n=5n-4$ ve $\dfrac{1}{k_1}+\dfrac{1}{k_2}+\dfrac{1}{k_3}+...+\dfrac{1}{k_n}=1$ eşitliğini pozitif tam sayılar için çözünüz. (Putnam Matematik Olimpiyatı)

$86)$ $(1+\dfrac{1}{x}).((1+\dfrac{1}{y}).(1+\dfrac{1}{z})=2$ eşitliğini pozitif tam sayılar için çözünüz. (İngiltere matematik olimpiyatı)

$87)$ Tüm $n$ tam sayılarını bulunuz öyle ki $(x+y+z)^2=nxyz$ pozitif tam sayılarda çözümlü olsun. (American Mathematically Reformulation)

$88)$ $ x^2+84x+2008=y^2$  eşitliğini sağlayan pozitif $x,y$ tam sayıları için $x+y$ kaçtır? $(AIME)$

$89)$ $A,B,C$ rakamlar olmak üzere $(100A+10B+C).(A+B+C)=2005$ eşitliğini sağlayan $A$ değeri kaçtır? (AMC $12A$ $2005$)

$90)$ $71p+1$ bir doğal sayının tam karesi olacak şekildeki tüm $p$ asallarını bulunuz. (Purple Comet MS $2011$)

$91)$ $p,q,r$ asal sayılar olmak üzere $pqr=7.(p+q+r)$ olduğuna göre $p+q+r$ kaçtır?

$92)$ $n-76$ ve $n+76$ aynı anda bir pozitif sayının kübü olacak şekildeki $n$  tam sayılarını bulunuz. (Purple Comet HS $2004$)

$93)$ $a$ ve $b$ pozitif tam sayıları için

$$\log _{2} (\log_{2^a}(\log_{2^b}2^{1000}))=0$$ eşitliğini sağlayan tüm $a+b$ değerlerinin toplamı kaçtır? (AIME-1 $2013$)

$94)$ $4^y-615=x^2$ eşitliğini sağlayan $(x,y)$ ikililerini bulunuz. (American Math League $2005-2006$)

$95)$ $2.(x^2+y^2)+x+y=5xy$  denkleminin tüm $(x,y)$ tam sayı ikililerini bulunuz. (Awesome Math Test A)

$96)$ $x^3+y^3=z^3$ denkleminin tam sayılardaki genel çözümünü bulunuz.

$97)$ $x_0,x_1,...,x_{2011}$ negatif olmayan tam sayılar olmak üzere $m^{x_0}=\sum_{k=1}^{2011} m^{x_k}$ olmasını sağlayan kaç $m$ pozitif tam sayısı vardır? (AIME $2011$)

$98)$ $m$ tam sayısı için $x^3-2011x+m$ ifadesinin $3$ tam sayı kökü $a,b,c$ olsun. Buna göre $\mid a \mid+\mid b \mid+\mid c \mid$ ifadesinin değeri kaçtır? (AIME $2011$)

$99)$ $x^2+1=2y^4$ eşitliğini tam sayılarda çözünüz.

$100)$ $x^4=y^2+z^2+4$ eşitliğinin tam sayılarda çözümü olmadığını gösteriniz.

$101)$ $3^k=m^2+n^2+1$ eşitliğinin pozitif tam sayılarda sonsuz çözümü olduğunu gösteriniz (St. Petersburg)

$102)$  $x^3+117y^3=5$ denklemini tam sayılarda çözünüz.

$103)$ $(x^2+x+1).(y^2+y+1)=z^2+z+1$ eşitliğini tam sayılar için çözünüz.

$104)$ $m^3+6m^2+5m=27n^3+9n^2+9n+1$ eşitliğinin tam sayılarda çözümlerini bulunuz.

$105)$ $1!+2!+3!+...+x!=y^2$ eşitliğini tam sayılarda çözünüz.

$106)$ $x^2+y^2=2z^2$ eşitliğini pozitif tam sayılarda çözünüz.

$107)$ $x^2+y^2=3z^2$ eşitliğini pozitif tam sayılarda çözünüz.

$108)$ $x^3+3y^3=9z^3$ eşitliğinin pozitif tam sayılarda çözümü olmadığını gösteriniz.

$109)$ $x^3+8x^2-6x+8=y^3$ eşitliğini negatif olmayan tam sayılar için çözünüz.

$110)$ $19x^3-84y^2=1984$ eşitliğini tam sayılarda çözünüz.

$111)$ Eğer $n=a^2+b^2+c^2$ eşitliğini sağlayan $a,b,c$ pozitif tam sayıları var ise $n^2=x^2+y^2+z^2$ eşitliğini sağlayan $x,y,z$ pozitif tam sayıları da olacağını gösteriniz.

$112)$ $n$ pozitif tam sayısı için  $3n+1$ ve $4n+1$ aynı anda tam kare olduğuna göre $56\mid n$ olduğunu gösteriniz.

$113)$ $5m^2-6mn+7n^2=1985$ olacak şekildeki $(m,n)$ pozitif tam sayı ikililerini bulunuz. (İzlanda Matematik Olimpiyatı $1985$)

$114)$ $x^2=2^n+3^n+6^n$ eşitliğini pozitif tam sayılarda çözünüz.

$115)$ $x^3+2y^3+4z^3-6xyz=1$ eşitliğini pozitif tam sayılarda çözünüz.

$116)$ $0\le x,y,z,t \le 10^6$ olmak üzere $x^2-y^2=z^3-t^3$ eşitliğinin  tam sayı çözümlerinin sayısı $M$ ve $x^2-y^2=z^3-t^3+1$ eşitliğinin tam sayı çözümlerinin sayısı $N$ olmak üzere $M>N$ olduğunu gösteriniz. (İzlanda Matematik Olimpiyatı (?))

$117)$ $x^2+5y^2=z^2$ eşitliğini tam sayılarda çözünüz.

$118)$ $x^5-y^2=4$ eşitliğinin tam sayılarda çözümünün olmadığını gösteriniz.

$119)$ $x,y$ pozitif tam sayıları için $4xy-x-y$ ifadesinin  tam kare olamayacağını gösteriniz.

$120)$ $x^3+y^3+z^3+t^3=1999$ eşitliğinin sonsuz çözümü olduğunu gösteriniz.

$121)$ Eğer $x,y$ pozitif tam sayılar ise $x^2-y^2$ ile $x^2+y^2$ ifadelerinin aynı anda tam kare olamayacağını gösteriniz.

$122)$ $4$ tane aritmetik dizi oluşturan tam kare tam sayı veriliyor. Hepsinin birbirine eşit olması gerektiğini gösteriniz.

$123)$ $x^4-y^4=z^2$ eşitliğinin pozitif tam sayılarda çözümünün bulunmadığını gösteriniz.

$124)$ $x^4-x^2y^2+y^4=z^2$ denkleminin tam sayılarda çözümlerini bulunuz.

$125)$ $x^4+x^2+y^2+y^4=z^2$ denkleminin tam sayılarda çözümlerini bulunuz.

$126)$ $\dfrac{1}{x^2}+\dfrac{1}{y^2}=\dfrac{1}{z^2}$ denkleminin pozitif tam sayılarda çözümlerini bulunuz.

$127)$ $x^6+x^3+x^3y+y=147^{157}$ ve $x^3+x^3y+y^2+y+z^9=157^{147}$ olduğuna göre $x,y,z$ sayılarının tamamının aynı anda tam sayı olamayacağını gösteriniz. (USAMO $2005$)

$128)$ $a,b,c$ pozitif tam sayıları $a,b$ aralarında asal ve $c$ de $a$ veya $b$ ile aralarında asaldır. Buna göre $x^a+y^b=z^c$ eşitliğinin sonsuz sayıda çözümü olduğunu gösteriniz. (İzlanda Matematik Olimpiyatı $1997$)

$129)$ $x$ bir rastgele bir rasyonel sayı olsun. Buna göre  $\dfrac{a^3+b^3}{c^3+d^3}=x$ eşitliğini sağlayan $(a,b,c,d)$ çözümü daima vardır. Gösteriniz. (İzlanda Matematik Olimpiyatı $1999$)

$130)$ $x^3+x^2y+xy^2+y^3=8.(x^2+xy+y^2+1)$ eşitliğini tam sayılarda çözünüz. (engel)

$131)$ $(5+3\sqrt{2})^m=(3+5\sqrt{2})^n$ eşitliğini sağlayan $m$ ve $n$ tam sayılarını bulunuz. (engel)

$132)$ $y^2+y=x^4+x^3+x^2+x$ eşitliğini tam sayılarda çözünüz. (engel)

$133)$ $\dfrac{x^n-1}{x-1}$ ifadesi tam kare olacak şekilde $(x,n)$ tam sayılarını belirleyiniz. (Nagel- Ljunggren)

$134)$ her $a_i \in \{-1,1 \}$  olmak üzere $a_1a_2+a_2a_3+...+a_{n-1}a_n+a_na_1=0$ ise $4\mid n$ olduğunu gösteriniz. (İzlanda Matematik Olimpiyatı)

$135)$ $p$ asal sayısı ve $a,b,c,d,n$ pozitif tam sayıları için $P=a^2+nb^2=c^2+nd^2$ ise $a=c$ dir gösteriniz. (engel)

$136)$ $m,n$ tam sayıları için $\mid 12^m-5^n\mid$ şeklinde yazılabilen en küçük pozitif tam sayıyı belirleyiniz.

$137)$ $p$ asal sayısı için $(p-1)!+1$  $p$ nin bir tam kuvveti olacak şekilde $p$ sayılarını bulunuz.

$138)$ $(2x^2+5)^2=20y^2+29$ eşitliğini sağlayan $x,y$ doğal sayılarını bulunuz. ($AOPS$)
« Son Düzenleme: Ağustos 31, 2023, 02:49:33 ös Gönderen: Metin Can Aydemir »
Bir matematikçi sanmaz fakat bilir, inandırmaya çalışmaz çünkü ispat eder.
    Boğaziçi Üniversitesi - Matematik

Çevrimdışı AtakanCİCEK

  • G.O Demirbaş Üye
  • ******
  • İleti: 264
  • Karma: +4/-0
  • Manisa
Ynt: Diyafont Denklemler Çalışma Soruları (70 Tane)
« Yanıtla #1 : Temmuz 18, 2019, 02:07:52 ös »
$1)$ 
 Denklemi $x^3+y^3+z^3-3xyz=81$ şeklinde yazalım. Çarpanlarına ayırarak $(x+y+z).(x^2+y^2+z^2-xy-xz-yz)=81$ şeklinde yazalım. $x<y<z$ ve $x,y,z$ pozitif tam sayılar olduğundan $x+y+z\ge6$ olmalıdır.$x+y+z|81$ olduğundan $x+y+z\in\{9,27,81\}$ olmalıdır.

$a1)$ $x+y+z=9$ , $xy+xz+yz=24$ olur.

$a2)$ $x+y+z=27$ , $xy+xz+yz=242$ olur.

$a3)$ $x+y+z=81$ olursa $xy+xz+yz\notin Z$ olur. $a3$ olamaz.

$x<y$ olduğundan $x-y<0$ olduğunu aklımızda tutalım.

      $a)$ $x+y+z=9$ ve $xy+xz+yz=24$ için

            $x^2+y^2+z^2=(x+y+z)^2-2.(xy+xz+yz)$ formülünü kullanalım.

      $x^2+y^2+z^2=81-48=33$ olur ve $z\le5$ olur.
      
      eğer $x=y=z$ olsaydı bile $3z=9$ olur bu nedenle $z>3$ olmalıdır. $z\in\{4,5\}$ olmalıdır.

         $1)$ $z=4$ ise $x+y=5$ olur. $(x+y).z+xy=24$

                 $xy=4$ bulunur. Buradan $(x-y)^2=(x+y)^2-4xy$ olduğunu kullanarak

             $x-y=-3$ bulunur. buradan $(1,4,4)$ çözümü gelir ancak $x<y<z$ sağlanmaz.
   
         $2)$ $z=5$ ise $x+y=4$ olur $(x+y).z+xy=24$

            $x-y=0$ bulunur $x=y$ $x<y$ ile çelişir.

      $b)$ $x+y+z=27$ ve $xy+xz+yz=242$ için
      $x^2+y^2+z^2=(x+y+z)^2-2.(xy+xz+yz)$ formülünü kullanalım.
      $x^2+y^2+z^2=729-2.242=245$ elde edilir. $z\le15$ olmalıdır.

      $x=y=z$ olsaydı bile $3z=27$ $z=9$ bu nedenle $z>9$ olmalıdır.

$z\in \{10,11,12,13,14,15\}$ olabilir.
\begin{equation*}
\begin{cases}
x+y=17,
\\
xy=72,
\\
z=10
\end{cases}
\end{equation*}
\begin{equation*}
\begin{cases}
x+y=16,
\\
xy=66,
\\
z=11
\end{cases}
\end{equation*}

$2.$ sistemde $16^2-4.66=(x-y)^2<0$ olduğundan tam sayı çözümü yoktur.

Geriye kalan yazılabilecek sistemlerde $(x+y)^2\le225$ ve $-4xy\le-240$ olduğundan

 $(x-y)^2\le0$ çelişkisi ortaya çıkar.
$1)$ sistemini çözelim
$(x-y)^2=17^2-4.72=1$ ve $x-y\le0$ olduğundan $x-y=-1$ olur.
\begin{equation*}
\begin{cases}
x+y=17,
\\
x-y=-1,
\\
z=10
\end{cases}
\end{equation*}

denklem sisteminin çözümü diyafont denklemimizin çözümüdür ve $(8,9,10)$ olarak bulunur.
« Son Düzenleme: Ağustos 02, 2019, 06:51:12 ös Gönderen: AtakanCİCEK »
Bir matematikçi sanmaz fakat bilir, inandırmaya çalışmaz çünkü ispat eder.
    Boğaziçi Üniversitesi - Matematik

Çevrimdışı AtakanCİCEK

  • G.O Demirbaş Üye
  • ******
  • İleti: 264
  • Karma: +4/-0
  • Manisa
Ynt: Diyafont Denklemler Çalışma Soruları (70 Tane)
« Yanıtla #2 : Temmuz 18, 2019, 02:08:47 ös »
$3)$ Denklemin her iki tarafına $1$ ekleyip denklemi düzenleyelim.

$y^2+1=x^3+8$

Şimdi $y^2+1$ için geçerli olan özellikler bulalım.

$y=2k$ olursa $y^2+1\equiv 1(mod4)$ olur.

$y=2k+1$ olursa $y^2+1 \equiv 2(mod4)$ olur. $(1)$

Şimdi $y$ nin $2$ dışında asal çarpanlarının tamamının $4k+1$ formunda olduğunu gösterelim.$(2)$

$p\mid y^2+1$ olsun. $y^2\equiv -1(modp)$ olur

$y^4\equiv1(modp)$

 $p\mid y^2+1$ olduğundan $(y,p)=1$ dir.

Fermat teoreminden $y^{p-1}\equiv1(modp)$

Buradan $4$'ün $p-1$ in en küçük katı olduğunu görmek mümkündür $4\mid p-1$ elde edilir.

Şimdi sorumuza geri dönelim.
$x=2m$ ,$m\in Z$
$y^2+1=(2k+2).(4k^2-4k+4)$  olur ve $(1)$ ile çelişir.

$x=2m+1$ , $m\in Z$ olsun. $x^2-2x+4\equiv3(mod4)$ olacağından ve $(2)$ den dolayı mümkün olmadığını söyleriz. Tam sayılarda çözümü yoktur.
Bir matematikçi sanmaz fakat bilir, inandırmaya çalışmaz çünkü ispat eder.
    Boğaziçi Üniversitesi - Matematik

Çevrimdışı AtakanCİCEK

  • G.O Demirbaş Üye
  • ******
  • İleti: 264
  • Karma: +4/-0
  • Manisa
Ynt: Diyafont Denklemler Çalışma Soruları (70 Tane)
« Yanıtla #3 : Temmuz 18, 2019, 02:09:56 ös »
$4)$
Verilen denklemin $k$ ve $m$ negatif olmayan tam sayıları için çözümü olduğunu kabul edelim.

$k!\equiv0(mod48)$ olması gerektiği barizdir. $k\ge6$ olur.
$3.5.7.8.9.....(k-1).k+1=(k+1)^m$ şeklinde denklem yazılabilir.$(k+1)^m$ nin çarpanlarına ayrılıp ayrılamayacağını araştıralım.

$k+1$ bileşik sayı olsun ve $k+1$ sayısının asal bölenlerinden biri $p$ olsun. $p\mid k+1$ olduğundan ve $p=k+1$ olamayacağından $p<k+1$ yani $p\le k$ elde edilir. sol tarafın tek olduğunu göstermek için $k\ge8$ olduğunu gösterelim.

$k=6$ için denklem $\frac{720+48}{48}=16=7^m$ olur çözümsüzdür.

$k=7$ için denklem $106=8^m$ olur çözümsüzdür. $k\ge8$ olur.

Buradan $p$ nin tek sayı olduğu ve $k$ dan küçük eşit olduğu da açık olduğundan $p\mid 3.5.7.8.....(k-1).k$ olduğundan ve $p\mid(k+1)^m$ olduğundan $p|(k+1)^m-3.5.7.8.....(k-1).k$ yani $p\mid1$ çelişkisi ortaya çıkar. $k+1$ bir asal sayıdır.

Wilson teoreminden $k!\equiv(p-1)!\equiv-1(modp)$  yani $p\mid k!+1$ olur.

Başlangıçtaki denklem $k!+48=48p^m$ denklemini $47=48p^m-k!-1$ şeklinde yazarsak sağ tarafın $p$ ye bölünür yani $p\mid47$ elde edilir. Buradan $p=47$ olduğunu buluruz. Denklemi $p=47$ için yazalım.

$46!+48=48.47^m$ olur $46!=48.(47^m-1)$ $46!$ in asal bölenlerinden $48$ ile aralarında asal olacak şekilde seçersek $13$ $17$ seçilebilir. $47^m\equiv1(mod13)$ ve $47^m\equiv1(mod17)$ olabileceğinden Fermat teoremi yardımıyla $m\ge48$ olur. 

$48.47^m\ge48.47^{48}>46!+48$ olduğu açıktır dolayısıyla tam sayı çözümü olamaz.
Bir matematikçi sanmaz fakat bilir, inandırmaya çalışmaz çünkü ispat eder.
    Boğaziçi Üniversitesi - Matematik

Çevrimdışı AtakanCİCEK

  • G.O Demirbaş Üye
  • ******
  • İleti: 264
  • Karma: +4/-0
  • Manisa
Ynt: Diyafont Denklemler Çalışma Soruları (70 Tane)
« Yanıtla #4 : Temmuz 18, 2019, 02:11:23 ös »
$5)$
$2^m-3^n=7$ ise $2^m\equiv1(mod3)$ ve $m=2x$ , $x\in Z^+$ için çözümü vardır. $m=2$ için $-3^n=3$ çözüm yoktur.

$m>2$  çift sayıları için $2^m\equiv0(mod8)$ dolayısıyla $3^n\equiv1(mod8)$ olmalıdır. dolayısıyla $n$ de çift olmalıdır. $n=2y$, $y\in Z^+$ vardır.

$2^{2x}-3^{2y}=7$

$(2^x-3^y).(2^x+3^y)=7$

$2^x+3^y\ge2$ olduğunu kullanarak sistemi yazalım.

\begin{equation*}
\begin{cases}
2^x+3^y=7,
\\
2^x-3^y=1
\end{cases}
\end{equation*}

$2^x=4$ ve $3^y=3$ bulunur. $x=2$ , $y=1$ yani $(m,n)=(4,2)$ elde edilir.
Bir matematikçi sanmaz fakat bilir, inandırmaya çalışmaz çünkü ispat eder.
    Boğaziçi Üniversitesi - Matematik

Çevrimdışı AtakanCİCEK

  • G.O Demirbaş Üye
  • ******
  • İleti: 264
  • Karma: +4/-0
  • Manisa
Ynt: Diyafont Denklemler Çalışma Soruları (70 Tane)
« Yanıtla #5 : Temmuz 18, 2019, 02:12:10 ös »
$6)$

$a^2=b.(b^{1998}-1)$ yazalım. $b\ge2$ için $(b,b^{1998}-1)=1$ olduğundan dolayı $b^{1998}-1$ ifadesi tamkare olmalıdır.

Fakat $b^{1998}$ tamkare olduğu için ve $1$ den büyük tamkare sayılar arasındaki fark 3'ten büyük olduğundan dolayı $b^{1998}-1$ tamkare olamaz.

$b\le-2$ de olamaz çünkü $a^2<0$ olur. $b\in{-1,0,1}$ elde edilir.

$1)$ $b=-1$ ise $a=0$

$2)$ $b=0$ ise $a=0$

$3)$ $b=1$ ise $a=0$

Çözümleri bulunur.
Bir matematikçi sanmaz fakat bilir, inandırmaya çalışmaz çünkü ispat eder.
    Boğaziçi Üniversitesi - Matematik

Çevrimdışı AtakanCİCEK

  • G.O Demirbaş Üye
  • ******
  • İleti: 264
  • Karma: +4/-0
  • Manisa
Ynt: Diyafont Denklemler Çalışma Soruları (70 Tane)
« Yanıtla #6 : Temmuz 18, 2019, 02:12:48 ös »
$7)$

$13^3>2001$ olduğundan sayılar $13$ ten küçüktür.

$8^3+8^3+8^3<2001$ olduğundan en az biri de $8$ den büyüktür.

Genelliği bozmadan $a\ge b\ge c$ alalım.

buradan $a\in 9,10,11,12$ olabilir.

$1)$ $a=9$ için $b^3+c^3=1272$ olur. $b=c=8$ olsa $1024<1272$ $b>8$ ve $a=9\ge b$ olduğundan $b=9$ olmak zorundadır. $1272-729=543$ olur $c^3=543$ denkleminin çözümü yoktur.

$2)$ $a=10$ için $b^3+c^3=1001$ olur. $b=c=7$ olsa $686<1001$ olacağı için $b\ge8$ $b\le10$ sağlanmalıdır. $b\in 8,9,10$ olabilir.

$b=8$ ise $c^3=489$ çözümü yoktur.

$b=9$ ise $c^3=272$ çözümü yoktur.

$b=10$ ise $c^3=1$ buradan $c=1$ olur.
 
$3)$ $a=11$ için $b^3+c^3=670$ olur. $b=c=6$ için $432<670$ olduğundan $b\ge7$ olmalıdır. $9^3=729$ olduğundan $b<9$ da olmaldıır. $b\in 7,8$ olabilir.

$b=7$ ise $c^3=327$ çözümü yoktur.      

$b=8$ ise $c^3=158$ olacağından çözüm yoktur.

$4)$ $a=12$ için $b^3+c^3=273$ olur. $b=c=5$ için $250<273$ olduğundan $b\ge6$ olmaldıır. $7^3=343$ olduğundan $b<7$ olur. $b=6$ olmalıdır.

$b=6$ ise $c^3=273-216=57$ çözümü yoktur.

Dolayısıyla denklemin $a\ge b\ge c$ için tek çözümü $(10,10,1)$ olduğundan çözüm sayısı bunların sıralanışları kadardır. $\frac{3!}{2!}=3$ olarak bulunur.
Bir matematikçi sanmaz fakat bilir, inandırmaya çalışmaz çünkü ispat eder.
    Boğaziçi Üniversitesi - Matematik

Çevrimdışı AtakanCİCEK

  • G.O Demirbaş Üye
  • ******
  • İleti: 264
  • Karma: +4/-0
  • Manisa
Ynt: Diyafont Denklemler Çalışma Soruları (70 Tane)
« Yanıtla #7 : Temmuz 18, 2019, 02:14:05 ös »
$8)$ Denklemi $\frac{1}{x}+\frac{1}{y}+\frac{1}{z}=\frac{3}{5}$ olarak yazalım.
 Genelliği bozmadan $1< x \le y \le z$ alalım.

$x\ge6$ için eşitsizlik oluşturalım. $1\ge \frac{1}{x} \ge \frac{1}{y} \ge \frac {1}{z}$ olduğundan $\frac{1}{x}+\frac{1}{y}+\frac{1}{z}\le \frac{1}{2}$  olduğundan

$\frac{1}{x}+\frac{1}{y}+\frac{1}{z}=\frac{3}{5}$ sağlanmaz. 

$1)$ $x=5$ için ise tek çözüm $x=y=z=5$ olmalıdır.

şimdi burada $x$ in değerini seçeceğimiz için $\frac{1}{y}+\frac{1}{z}=\frac{a}{b}$ olduğunu kullanıp bir şeyler elde edelim.

öncelikle denklemi $ayz=by+bz$ şeklinde yazalım. Ardından $ayz-bz=by$, $z.(ay-b)=by$  $z=\frac{by}{ay-b}\in Z$ elde edilir.

$\frac{aby}{ay-b}\in Z$ de olması gerekir. Hatta $\frac{aby}{ay-b}+\frac{-aby+b^2}{ay-b} \in Z$ de sağlanmalıdır. Buradan $\frac{b^2}{ay-b}\in Z $ $(1)$  elde edilir.



$2)$ $x=4$ için ise $\frac{1}{y}+\frac{1}{z}=\frac{7}{20}$ olduğunu $(1)$ e uygulayalım.

$\frac{400}{7y-20}\in Z$ olduğunu kullanırsak ve  benzer şekilde $\frac{400}{7z-20}\in Z$ olduğundan ve $y\le z$ olduğundan $7y-20$ için  $\sqrt{400}$ e kadarki çarpanlarına bakmamız yeterlidir.

$7y-20$ $\in$ $A$ ,  $A\in1,2,4,5,8,10,16,20$ olmalıdır. $A$ kümesinin bir elemanı $a$ için  $7y\in 20+a$ olduğundan $a\equiv1(mod7)$ olacak şekilde seçilmelidir.

dolayısıyla $a$ nın alabileceği değerler $1,8$ dir.  Yani $y\in 3,4$ olur. $x\le y$ olduğundan yalnızca $y=4$ olmalıdır. $\frac{7}{20}-\frac{5}{20}= \frac{1}{10}$ olduğundan  $z=10$ bulunur. Çözüm $(4,4,10)$ olarak bulunur.



$3)$ $x=3$ için ise $\frac{1}{y}+\frac{1}{z}=\frac{4}{15}$ elde edilir. $(1)$ gereğince $\frac{225}{4y-15}\in Z$  olduğunu kullanıp $4y-15$ in $\sqrt{225}=15$ e kadar (dahil) olacak şekilde çarpanlarıdır. $4y-15\in 1,3,5,9,15$ olur. $y\in 4,5,6$ olduğunu görebiliriz.

$y=4$ için $z=60$ olur,

$y=5$ için $ z=15$ olur.

$y=6$ için $ z=10$ olur.




$4)$  $x=2$ için ise $\frac{1}{y}+\frac{1}{z}=\frac{1}{10}$  olur ve $(1)$ gereğince $\frac{100}{y-10}\in Z$ olmalıdır. $100$ un $10$ a kadar çarpanları $y-10$ un alabileceği değerlerdir. $y\in 11,12,14,15,20$ olabilir.

$y=11$ için $z=110$ olur.

$y=12$ için $z=60$ olur.

$y=14$ için $z=35$ olur.

$y=15$ için $z=30$ olur.

$y=20$ için $z=20$  olur.
   
$5)$ $x=1$ olamaz çünkü $\frac{1}{y}+\frac{1}{z}\le 0$ olur.

Gerçel çözümlerinin sayısı bu çözümlerin permütasyonları kadardır.

$3!+3!+3!+3!+\frac{3!}{2!}+3!+3!+3!+\frac{3!}{2!}+\frac{3!}{3!}=49$ olarak bulunur.
Bir matematikçi sanmaz fakat bilir, inandırmaya çalışmaz çünkü ispat eder.
    Boğaziçi Üniversitesi - Matematik

Çevrimdışı AtakanCİCEK

  • G.O Demirbaş Üye
  • ******
  • İleti: 264
  • Karma: +4/-0
  • Manisa
Ynt: Diyafont Denklemler Çalışma Soruları (70 Tane)
« Yanıtla #8 : Temmuz 18, 2019, 02:17:39 ös »
$9)$

$Ilk$ $Yol$


 
$(x-y).(x^2+xy+y^2)=xy+61$ 
\begin{equation*}
\begin{cases}
x-y=a,
\\
xy=b
\end{cases}
\end{equation*}

$a.(a^2+3b)=b+61$ $b>0$ olduğundan $a>0$ da olmalıdır.

$a^3+3ab=b+61$

$a^3-61=b.(1-3a)$

$b=\frac{a^3-61}{1-3a}>0$  olduğunu kullanırsak  $a^3<61$ elde edilir.  $a\in 1,2,3$ olmalıdır.

$a=1$ ise $b=30$

$a=2$ ise $b\notin Z$

$a=3$ ise $b\notin Z$

Yani $x-y=1$ $xy=30$ elde ederiz  $(x+y)^2=(x-y)^2+4xy=121$ yani $x+y=11$ olur. buradan denklemin çözümü

\begin{equation*}
\begin{cases}
x=6,
\\
y=5
\end{cases}
\end{equation*}

olarak bulunur.

$Ikinci$ $Yol$

Denkleme bakıldığında $x-y>0$ olduğu açık olduğundan $x=y+z$ ,$z\in Z^+$ dönüşümü yapılabilir.

$(y+z)^3-y^3=y.(y+z)+61$

$(3z-1).y^2+(3z^2-z).y+z^3-61=0$
 
ve $z\ge1$ olduğundan $3z-1\in Z^+$ hatta $3z^2-z\in Z^+$ olduğu da yazılır.Tüm terimlerin katsayıları denklemin pozitif sayılarda çözümü olamaz. $z^3<61$ olduğunu yine söyleriz.

$x-y=z\in 1,2,3$ olduğunu yine elde ederiz.
Bir matematikçi sanmaz fakat bilir, inandırmaya çalışmaz çünkü ispat eder.
    Boğaziçi Üniversitesi - Matematik

Çevrimdışı AtakanCİCEK

  • G.O Demirbaş Üye
  • ******
  • İleti: 264
  • Karma: +4/-0
  • Manisa
Ynt: Diyafont Denklemler Çalışma Soruları (70 Tane)
« Yanıtla #9 : Temmuz 18, 2019, 02:18:58 ös »
$10)$

$y-x=a$  $9x.(x+a)+127=(x+a)^3-x^3=a^3+3a^2x+3ax^2$

$(9-3a).x^2+(9a-3a^2).x+127-a^3=0$  $a=3$ olmadığı açık olduğundan

$x^2+ax+\frac{127-a^3}{9-3a}=0$

$9-3a\mid127-a^3$ ü $3-a\mid 127-a^3$ için çözelim.
Basit bir polinom bölmesiyle
$\frac{127-a^3}{3-a}=a^2+3a+9+\frac{100}{3-a}$ olur. buradan

 $a-3\in \{-100,-50,-25,-20,-10,-5,-4,-2,-1,1,2,,4,5,10,20,25,50,100\}$ olabilir. $3\mid 127-a^3$  ten dolayı $a\equiv 1(mod3)$ olur.

 $a\in \{-97,-47,-22-17,-7,-2,-1,1,2,4,5,7,8,13,23,28,53,103\}$  olur $a\equiv1(mod3)$ ten dolayı $a\in\{-47,-17,-2,1,4,7,13,28,103\} $ olabilir.

$a=-47$ için $x^2-47x+693=0$ olur $\bigtriangleup<0$ olduğundan çözüm yoktur.

$a=-17$ için $x^2-17x+84=0$ olur. $\bigtriangleup<0$ olduğundan çözüm yoktur.

$a=-2$ için $x^2-2x+9=0$ olur. $\bigtriangleup<0$ olduğundan çözüm yoktur.

$a=1$ için $x^2+x+21=0$ olur. $\bigtriangleup<0$ olduğundan çözümü yoktur.

$a=4$ için $x^2+4x-21=0$ olur. $x=-7$ ve $x=3$ çözümlerini buluruz. $(-7,-3)$ ve $(3,7)$ çözümlerdir.

$a=7$ için $a=x^2+7x+18=0$ olur. $\bigtriangleup<0$ olduğundan çözüm yoktur.

$a=13$ için $x^2+13x+69=0$ olur. $\bigtriangleup<0$ olduğundan çözüm yoktur.

$a=28$ için $x^2+28x+291=0$ olur. $\bigtriangleup<0$ olduğundan çözüm yoktur.

$a=103$ için $x^2+103x+3642=0$ $\bigtriangleup<0$ olduğundan çözüm yoktur. 

Dolayısıyla denklemin tam sayılarda $2$ çözümü vardır.
Bir matematikçi sanmaz fakat bilir, inandırmaya çalışmaz çünkü ispat eder.
    Boğaziçi Üniversitesi - Matematik

Çevrimdışı AtakanCİCEK

  • G.O Demirbaş Üye
  • ******
  • İleti: 264
  • Karma: +4/-0
  • Manisa
Ynt: Diyafont Denklemler Çalışma Soruları (70 Tane)
« Yanıtla #10 : Temmuz 18, 2019, 02:21:41 ös »
$11)$

$n^3+7n-133=x^3,x\in Z^+$ şeklinde denklemi yazalım.

$n.(n^2+7)-133$ ifadesi $n<5$ için pozitif tam sayı olamaz. $n\ge5$ olmalıdır.

$7n-133=x^3-n^3$

$n=19$ için $x=19$ olacağından  $(19,19)$ çözümü olur. Şimdi $n>19$ ve $n<19$ durumlarını inceleyelim.

$1)$ $n>19$  $x^3>n^3$ olduğundan $x=n+d,d\in Z^+$ dönüşümü yapabiliriz.

$3d.n^2+(3d^2-7).n+d^3+133=0$  $n\in Z^+$ olabilimesi için $3d^2-7<0$ olmalıdır. $d^2<\frac{7}{3}$ yani $d=1$  olmalıdır.

$d=1$ olursa $3n^2-4n+134=0$  olur ve $\bigtriangleup<0$ olduğundan reel çözümü yoktur.

$2)$ $n<19$ için bakalım. $n^3>x^3$ olduğundan $n=x+y$ , $y\in Z^+$ dönüşümü yapabiliriz.

$7x+7y-133=x^3-(x+y)^3=-3x^2y-3xy^2-y^3$ olur.

$3y.x^2+(3y^2+7).x+(y^3+7y-133)=0$ elde edilir.

$y^3+7y-133\ge0$ olduğunda tüm katsayılar pozitif  veya nötr olacağından Descartes işaret değişim kuralı gereğince Pozitif sayılarda çözümü yoktur. $y^3+7y-133<0$ yani $y\le5$ olur.

$y=1$ ise $3x^2+10x-125=0=(3x+25).(x-5)$ olur.  $x=5$ ,$y=1$ yani $n=6$ olur.

$y=2$ ise $6x^2+19x-11=0=(x-3).(6x+37)$ olur. $x=3$ ,$y=2$ yani $n=5$ olur.

$y=3$ ise $9x^2+34x-85=0$ olur. $\bigtriangleup=4216$ yani $\bigtriangleup$ tam kare olmadığından tam sayı çözümü yoktur.

$y=4$ ise $12x^2+55x-41=0$ $\bigtriangleup=4993$ yani $\bigtriangleup$ tam kare olmadığından tam sayı çözümü yoktur.

o halde $n$ sayısının alabileceği değerler $n\in \{5,6,19\}$ olur.
« Son Düzenleme: Temmuz 25, 2019, 02:41:45 ös Gönderen: AtakanCİCEK »
Bir matematikçi sanmaz fakat bilir, inandırmaya çalışmaz çünkü ispat eder.
    Boğaziçi Üniversitesi - Matematik

Çevrimdışı AtakanCİCEK

  • G.O Demirbaş Üye
  • ******
  • İleti: 264
  • Karma: +4/-0
  • Manisa
Ynt: Diyafont Denklemler Çalışma Soruları (70 Tane)
« Yanıtla #11 : Temmuz 18, 2019, 02:22:26 ös »
$12)$ $n^2-19n+99=x^2$ , $x\in Z^+$ olsun.

$n_{1,2}=\frac{19\pm\sqrt{361-4.(99-x^2)}}{2}$ tam sayı olabilmesi için  $361-4.(99-x^2)$ tam kare  yani $361-396+4x^2=4x^2-35$ tam kare olmaldıır. $4x^2-35=y^2,y\in Z^+$ alabiliriz.

$4x^2-y^2=35$ $(2x-y).(2x+y)=35$ $(2x+y)>0$ olduğundan $(2x-y)>0$ aynı zamanda $2x-y<2x+y$ olduğundan buna uygun denklem sistemlerini yazalım.

\begin{equation*}
\begin{cases}
2x+y=35,
\\
2x-y=1
\end{cases}
\end{equation*}

\begin{equation*}
\begin{cases}
2x+y=7,
\\
2x-y=5
\end{cases}
\end{equation*}

bu denklem sistemlerinden $y=17$ ve $y=1$  çözümleri elde edilir.

$n_{1,2}=\frac{19\pm\sqrt{361-4.(99-x^2)}}{2}=\frac{19\pm y}{2}$ ifadesinde yerine koyalım. 

$\frac{19\pm 17}{2}+\frac{19\pm 1}{2}=38$ elde edilir.
« Son Düzenleme: Ocak 02, 2020, 07:36:26 ös Gönderen: metonster »
Bir matematikçi sanmaz fakat bilir, inandırmaya çalışmaz çünkü ispat eder.
    Boğaziçi Üniversitesi - Matematik

Çevrimdışı AtakanCİCEK

  • G.O Demirbaş Üye
  • ******
  • İleti: 264
  • Karma: +4/-0
  • Manisa
Ynt: Diyafont Denklemler Çalışma Soruları (70 Tane)
« Yanıtla #12 : Temmuz 18, 2019, 02:24:23 ös »
$17)$

İlk Yol

$a^3-b^3-c^3=3abc$ ise  $ a^3=b^3+c^3+3abc $ ve $a^3>b^3+c^3$ elde edilir. $b$ ve $c$ simetrik olduğundan genelliği bozmadan $a>b\ge c$ olmalıdır. $a>b\ge c$ eşitsizliğinden $3a>a+b+c$ $6a>a^2$ elde edilir. yani $a<6$ eşitsizliğini elde ederiz.

$a^2=2.(a+b+c)$ olduğundan dolayı $a\equiv 0(mod2)$ olacağından  $a=2$ veya $a=4$ olmalıdır.

$1)$ $a=2$ için

$4=2.(2+b+c)$ yani $b+c=0$ elde edilir. Fakat $b,c\in Z^+$ olduğundan $b+c\ge2$ olmaldır. Dolayısıyla $a=4$ olmalıdır.

$2)$ $a=4$ ise $b+c=4$ elde edilir. aynı zamanda $a^3-b^3-c^3-3abc$ nin çarpanlarından biri $(a-b-c)$ olduğundan dolayı $1.$ denklemin doğru olmasını sağlar.

o halde $(4,1,3),(4,2,2),(4,3,1)$  şeklinde 3 adet çözüm bulunur . 

İkinci Yol

$a^3-b^3-c^3-3abc=0=(a-b-c).(a^2+b^2+c^2+ac+ab-bc)$

$1)$  $a=b+c$ ve $a^2=2.(a+b+c)$ bu nedenle $a^2-4a=0$ ve $a=4$ ve $b,c \in Z^+$ buradan gelen çözümler $(4,3,1),(4,2,2),(4,1,3)$

$2)$
\begin{equation*}
\begin{cases}
a^2+b^2+c^2=-ac-ab+bc,
\\
a^2=2.(a+b+c)
\end{cases}
\end{equation*} 

$n\in Z$ için $x_1,x_2,x_3,...,x_n\in R$ ve $y_1,y_2,y_3,...,y_n\in R$ için 

$\mid x_1.y_1+x_2.y_2+...+x_n.y_n \mid \le \sqrt{x_1^2+x_2^2+...+x_n^2}.\sqrt{y_1^2+y_2^2+...+y_n^2}$

Eşitlik yalnızca $\frac{x_1}{y_1}=\frac{x_2}{y_2}=...=\frac{x_n}{y_n}$ ile mümkündür (Cauchy-Schwarz)

$(-a).c+(-b).a+c.b\le \sqrt{a^2+b^2+c^2}. \sqrt{a^2+b^2+c^2}$

$-ac-ab+bc\le a^2+b^2+c^2$  olduğundan

$\frac{-a}{c}=\frac{c}{b}$ olması gereklidir.

$c^2=-ab$ yani  $c^2<0$ mümkün değildir..

Çözümleri $(4,3,1) ,(4,2,2),(4,1,3)$  olarak bulunur .

Üçüncü Yol

$a^3-b^3-c^3-3abc=0=(a-b-c).(a^2+b^2+c^2+ac+ab-bc)$

$1)$  $a=b+c$ ve $a^2=2.(a+b+c)$ bu nedenle $a^2-4a=0$ ve $a=4$ ve $b,c \in Z^+$ buradan gelen çözümler $(4,3,1),(4,2,2),(4,1,3)$

$2)$
\begin{equation*}
\begin{cases}
a^2+b^2+c^2=-ac-ab+bc,
\\
a^2=2.(a+b+c)
\end{cases}
\end{equation*} 

$(a-b-c)^2=-3ac-3ab+3bc$

$(a+b)^2+(a+c)^2+(b-c)^2=0$  olduğundan $a+b=0$ olmalıdır fakat mümkün değildir.
« Son Düzenleme: Temmuz 25, 2019, 02:42:52 ös Gönderen: AtakanCİCEK »
Bir matematikçi sanmaz fakat bilir, inandırmaya çalışmaz çünkü ispat eder.
    Boğaziçi Üniversitesi - Matematik

Çevrimdışı AtakanCİCEK

  • G.O Demirbaş Üye
  • ******
  • İleti: 264
  • Karma: +4/-0
  • Manisa
Ynt: Diyafont Denklemler Çalışma Soruları (70 Tane)
« Yanıtla #13 : Temmuz 18, 2019, 02:25:14 ös »
$18)$

Denklemi $2n^3-m^3-mn^2=11$ şeklinde düşündüğümüzde Soldaki terimlerin dereceleri $3$ olduğundan ve katsayıları toplamı $2-1-1=0$ olduğundan dolayı  $m=n-k$ , $k\in Z$ dönüşümü yapalım.($m=n+k$ dönüşümü de uygundur.)

$2n^3-n^3+3n^2k-3nk^2+k^3=n^3-kn^2+11$

$4k.n^2-3k^2.n+k^3-11=0$ denklemi elde edilir.

$\bigtriangleup=(-3k^2)^2-4.(4k).(k^3-11)=-7k^4+176k$ olur.

$k<0$ olursa $-7k^4<0$ ve $176k<0$ olduğundan $\bigtriangleup<0$ olur $n$ tam sayısı bulunamaz.

$k=0$ olursa başlangıçtaki denklem $2n^3-n^3-n^3=11$ olur $0=11$ çelişkisi ortaya çıkar. $k\in Z^+$ olması gerektiği bulunur.

$k>0$ için $\bigtriangleup=k.(-7k^3+176)>0$ eşitsizliğini çözelim.  $-7k^3+176>0$ yani $7k^3<176$ olur. bu eşitsizliğin çözümü $k<3$ şeklindedir.

$1)$ $k=1$ ise $4n^2-3n-10=(4n+5).(n-2)=0$ yani $n=2$ elde edilir.

$2)$ $k=2$ ise $8n^2-12n-3=0$ denkleminin $\bigtriangleup$ değeri $240$ olduğundan yani tamkare olmadığından tam sayı çözümü yoktur.

Buradan denklemin tek çözümü $(m,n)=(1,2)$ olarak bulunur. 
Bir matematikçi sanmaz fakat bilir, inandırmaya çalışmaz çünkü ispat eder.
    Boğaziçi Üniversitesi - Matematik

Çevrimdışı AtakanCİCEK

  • G.O Demirbaş Üye
  • ******
  • İleti: 264
  • Karma: +4/-0
  • Manisa
Ynt: Diyafont Denklemler Çalışma Soruları (70 Tane)
« Yanıtla #14 : Temmuz 18, 2019, 02:41:05 ös »
$20)$
İlk çözümdeki katkıları için metonstere teşekkür ediyorum.


İlk Yol

Soruda istenen ifadeye $a$ diyelim.

$\frac{13^m+p.2^n}{13^m-p.2^n}=a$ diyelim. İçler dışlar çarpımı yapıp düzenlersek ifademiz

$(1+a).p.2^n=13^m.(a-1)$     $4$ bilinmeyenli diyafont denklemi elde edilir. Bu ifadeden yola çıkarak

$p.2^n=13^m.\frac{a-1}{a+1}$ haline gelir. Sağ taraf birlikte düşünüldüğünde tamsayı olmalıdır.

$p.2^n=13^m-\frac{2.13^m}{a+1}$ haline gelir. Buradan

$a+1=2$,$a+1=1$,$a+1=-1$,$a+1=-2$,$a+1=13^k$, $ a+1=-13^k$,$a+1=2.13^k$,$a+1=-2.13^k$ ;($k\le m$)  eşitlikleri elde edilir. Bu ifadelerden eşitliği mümkün olan durumları bulalım.

Eşitliğin sol tarafı $0(mod2)$ olduğundan eşitliğin sağ tarafında da ifade çift olmalıdır.  $13^m\equiv1(mod2)$ olduğundan $\frac{2.13^m}{a+1}\equiv1(mod2)$ olmalıdır. Bunun için $a+1$ çift olmalıdır. $a>0$ olduğu da göz önüne alınırsa

$a+1=2$ veya $a+1=2.13^k$,$k\le m$  olabilir.

$1)$ $a+1=2$ olması durumunda $2.p.2^n=0$ olur ve $2^n>0$ ve $p>0$ olduğundan mümkün değildir.

$2)$ $a+1=2.13^k$,$k\le m$ olması durumunda $a=2.13^k-1$  olur ve

$p.2^n=13^m-13^{m-k}$ elde edilir. $m\neq k$ için ifade  $13^g.(13^t-1)$,$g\in N$ ve $t\in N$ olması gerektiğinden daima $12$ ve $13$ ile bölünür. dolayısıyla $p$ asal olamaz. $m=k$ olmalıdır.

$p.2^n=13^m-1$ Bu ifadenin içinde daima $12$ çarpanı bulanacağından ve başka tek çarpan daha  bulunacağından $m>1$ için $p$ asal olamaz

İddia: $m>1$ için $p.2^n=13^m-1$ ifadesine $13^m-1$ sayısının $2$ asal carpanı olan $p$ asal sayısı yoktur.

ispat:

Öncelikle $13^m-1$ sayısının her $m$ için $2$ ve $3$ e bölünebileceği aşikardır. Eğer $m=2n$ ise

\begin{equation*}
13^{2n}-1\equiv 169^n-1\equiv 1^n-1\equiv 0 (mod 7)
\end{equation*}

olduğundan $m$ çift ise $42$ ile bölünür ve en az $3$  asal böleni olmuş olur.

Eğer $m=2n+1$ ise farzedelim ki $13^m-1$ sayısı sadece $2$ ve $3$ ile bölünsün.

\begin{equation*}
13^{2n+1}-1\equiv 13\cdot 169^n-1\equiv 5\cdot 1^n-1\equiv 4(mod8)
\end{equation*}

olduğundan $13^m-1$ sayısı $4$ ile bölünür fakat $8$ ile bölünmez .Dolayısıyla $13^m-1=4.3^a$ formatında olmaldır. Kuvvet Kaydırma Teoremi uygulayalım,

\begin{equation*}
v_{3}(4\cdot 3^a)=a=v_{3}(13^m-1)=v_{3}(13-1)+v_{3}(m)=v_{3}(m)+1
\end{equation*}

Buradan $v_3(m)=a-1$ bulunur. Dolayısıyla $m=k.3^{a-1}$ formatında olmalıdır.

\begin{equation*}
4\cdot 3^a=13^m-1=13^{k\cdot 3^{a-1}}-1\geq 13^{3^{a-1}}-1
\end{equation*}

$a>1$ olduğunda $13^{3^a-1}-1>4.3a$ olacağı aşikardır. Dolayısıyla kabulumuz yanlıştır. Yani $m>1$ için $13^m-1$ sayısının en az $3$ asal böleni vardır.

 Bu nedenle $m>1$ için $p.2^n=13^m-1=2^n.3^{d_1}.q^{d_2}$ şeklinde yazılabileceği için $p$ asal olamaz.


$m=1$ alınmalıdır.

$p.2^n=12$ olur. $n=2$ ve $p=3$ elde edilir.

$(m,n,p)=(1,2,3)$ elde edilir.

İkinci Yol

 $a=\frac{13^m+p.2n}{13^m-p.2^n}=1+\frac{p.2^{n+1}}{13^m-p.2^n}$
$13^m-p.2^n$ olduğundan $p = 13$ olursa $p$ ye
sadeleştirdikten sonra payda tek, pay çift oluyor. $p\neq13$ ise payda $p$ ye
bölünmez ve tektir. Yani $a = 13m - p.2^n = 1$ dir. $(mod 3)$ ten p = 3
olur. $m = 1$ ve $ n = 2$  sağlar. $m \ge 2$ ve $n \ge 3$ ise
$(mod8)$ ifadesinden $m$ çift olmalıdır. Fakat bu durumda da $(mod7)$ den çelişki gelir.
« Son Düzenleme: Ağustos 02, 2019, 02:58:26 ös Gönderen: AtakanCİCEK »
Bir matematikçi sanmaz fakat bilir, inandırmaya çalışmaz çünkü ispat eder.
    Boğaziçi Üniversitesi - Matematik

 


Sitemap 1 2 3 4 5 6 7 8 9 10 11 12 13 14 15 16 17 18 19 20 21 22 23 24 25 26 27 28 29 30 31 32 33 34 35 36 37 
SimplePortal 2.3.3 © 2008-2010, SimplePortal